How can we test the holographic principle and nonlocality in quantum mechanics?

In summary, the conversation discusses the topic of nonlocality and its relation to quantum mechanics and special relativity. The book, The Holographic Universe by Michael Talbot, is mentioned as a source of information for this topic. The conversation also mentions the book The Dancing Wu Li Masters by Gary Zukav as a good starting point for those who are not well-versed in physics. The conversation then delves into a discussion about Bohm's interpretation of nonlocality and how it relates to special relativity. The concept of entanglement is also brought up as a possible evidence for nonlocality. The conversation ends with a debate about the consistency of quantum theory with special relativity and the existence of
  • #176
Careful said:
** if you try to construct a non-vague theory by simply getting rid of the second kind of dynamics, then the theory no longer predicts that measurments have definite outcomes, which is contrary to fact.) **

Of course that is not true, you have to change the Schroedinger equation too. By the way, perhaps this is not an issue amongst Bohm lovers, but some others might think differently.

So you're talking about GRW? It's a fine theory -- probably the second best available option.


** Bohm's theory solves the measurement problem unambiguously. It doesn't give two different dynamical rules. There is just one kind of dynamics, and everything (even the "stuff" that measurement apparatuses are made of) are all treating on an equal footing. And the theory actually predicts that measurements have outcomes -- pointers on detectors are made of particles, and these always end up in some definite place (because they're always at some definite place). **

Huh ?? The issue is that you simply don't KNOW where the particle is although it is somewhere and following a definite trajectory. So, you still have to indicate when it is that you ``percieve'' it at some spot and consequently generate a new wavepackage to guide it.

I don't follow the last part. There is no new dynamics for measurements in BM (no "new wavepackages" need to be "generated").



Moreover, Copenhagen also has only one DYNAMICAL rule, the projection postulate has nothing to do with dynamics.

So then why don't people simply drop the collapse rule and formulate the theory with sch-evolution only? Oh right, because of the measurement problem.

Let me put it less sarcastically: if there's some aspect of the mathematics which a theory requires in order to make correct contact with experiment, that bit of mathematics is dynamics. If you don't think it is, you are free to construct a new theory with a simpler dynamics -- ie, which simply never mentions the thing you think isn't real, isn't dynamical.



Me think that you cannot read between the lines.

Sorry, I didn't see anything written there.


Our ignorance is of course in the probability density of the wave function (we do not know where the particle is) and what makes it so strange is that this entity governs the dynamics of the particle through the quantum potential. Hence, the fact that this particle (or another particle in the same sample) *could* be somewhere else (in the future) is actually influencing the motion of the particle under consideration (now). If you don't find that strange, then I don't know what is to you.

If I understand your worry here, it's that in BM the wf is "merely epistemological" in that its only role is to provide a probability distribution for which positions/trajectories are actually realized. But this is just based on a confusion. The wf is not merely epistemological in BM. It is physically real, as real as the particles and their positions. What exists is particles being guided along their trajectories by the wf. Any epistemological character the wf has is *secondary* and, indeed, must be *derived* from its ontological/dynamical character. (But no worry, it can be so derived.)
 
Physics news on Phys.org
  • #177
Farsight said:
People are so steeped in their axiomatic particles that they invent parallel universes and even time travel to explain them. They just won't see the solution: the quanta you are dealing with ARE NOT PARTICLES. A quantum is NOT all in one place. It is NOT a point. A gallon is not a particle. A coulomb is not a particle. A quantum IS NOT LOCAL.

But people don't listen, and instead we have a god damn crackpot theological "debate" going round in circles for fifty years pretending to be physics. Absolutely tragic.

No Farsight, what is crackpottish is the following; a Bohmian coming to you and telling :
(a) A particle is not a point but we enrich reality by putting in point particles :rofl:
(b) Bohmian mechanics is the right way to see reality while NO attempt to connect with the reality of GR is made at all (!)
(c) Actually, the interpretational problems I gave you before *are* due to a hidden form of super-determinism, reversing arrow of time and so on. I would expect any theory which claims to be somehow more *real* to make such link EXPLICIT in terms of (local) physical processes (and no, Bell does not forbid that !).

The problem with Bohmpkes (like we say in dutch) and all these other super quantum oriented people writing about these issues, is that they forget there is other physics out there.

Careful
 
Last edited:
  • #178
**More precisely, I mean to say that if it were not for wave collapse, none of the odd behavior of QM would exist.**

Wel, that is no solution right ?? You still did not say where and when this transformation happens, and what the correct mechanism is to trigger it.

However (!), I do feel something for your proposal, it at least tries to adress QM in a more realistic way (but then in 5 dimensions). In the same way, I can say something good about GRW which adresses the problem of emergent classicality by putting in reasonable collapse-time and length scales, the same goes for the proposal by Penrose. The problem lies of course in the relativistic invariance, and although these approaches certainly go in the good direction - there is still lacking some deeper insight. I don't know whether a superdeterministic, local Planck scale proposal will make it but (!) (a) it is not ruled out (b) it is the most controlled, *least* conspirational approach to QG one can imagine (the world of LQG and superstring theory is much wilder) (c) emergent phenomena in the LQG like approaches certainly have to pass a similar scaling procedure. Therefore, it seems to me the most logical starting point to try the ``most simple'' idea first ...



Careful
 
Last edited:
  • #179
Hurkyl said:
I agree with the sentiment -- but I think you are at least as guilty of this as Careful.
I don't think so - I have no sentiments to determinism whatsoever. What I *do* have sentiments to is that QM lovers who have never thought about QG and what to do with QM on much higher energy scales, do not
(a) realize that a straightforward application of QM would make our macroworld a priori much more conspirational than a deterministic scenario would.
(b) recognize that Bell's assumptions are like a swiss cheese.

Moreover, as I said before, none of these interpretations/modifications do really solve something.
The problem as I see it, is that the ``new'' generation has given up the goal of *understanding* nature and this for the wrong reasons (expressed by some people in the past).

Careful
 
Last edited:
  • #180
**So you're talking about GRW? It's a fine theory -- probably the second best available option. **

GRW is non local, so why would I like it (although it adresses some of the issues I worry about and I have considered it previously)?

**
I don't follow the last part. There is no new dynamics for measurements in BM (no "new wavepackages" need to be "generated").
**

But what is observation then ? At some point you have to give the particle a CLASSICAL meaning, you should make it interact with classical fields (while the *wave function* does not interact with the latter); For example in the double slit experiment, there is nothing in the wave function which tells you that the particle will end up at any particular place on the screen, the only way to avoid this is to add an extra ingredient like the reduction (which tells you that somewhere it will appear, be we don't know where and when - and if it happens then we have to use an appropriate projection operator) or a *distinguished* classical element which undergoes *more* interactions than the wave does. This split again seems to be entirely arbitrary, so there really is no unified dynamics.


**
So then why don't people simply drop the collapse rule and formulate the theory with sch-evolution only? Oh right, because of the measurement problem. **

:rolleyes: First of all, Vanesch would protest here. Second, most people do see that Bohmian mechanics gives the problem a more reasonable face, but since no real solution is presented and the conflict with GR and SR is blatant, most see it as window dressing.


**If I understand your worry here, it's that in BM the wf is "merely epistemological" in that its only role is to provide a probability distribution for which positions/trajectories are actually realized. But this is just based on a confusion. The wf is not merely epistemological in BM. It is physically real, as real as the particles and their positions. What exists is particles being guided along their trajectories by the wf. **

No, you still do not understand what I say (although Farsight seems to) : what you say is moreover trivial and can be found in any textbook. The problem is not the fact that I can see the wavefunction in two ways (that is just cheap blablaba), the issue is that it is very hard to believe that this non-local quantity which contains as well information about the future and the past can serve as a physical guidance mechanism; without people giving a more in depth mechanism which is closer to GR as why this should be so.


Careful
 
Last edited:
  • #181
Careful said:
But what is observation then ? At some point you have to give the particle a CLASSICAL meaning, you should make it interact with classical fields (while the *wave function* does not interact with the latter); For example in the double slit experiment, there is nothing in the wave function which tells you that the particle will end up at any particular place on the screen, the only way to avoid this is to add an extra ingredient like the reduction (which tells you that somewhere it will appear, be we don't know where and when - and if it happens then we have to use an appropriate projection operator) or a *distinguished* classical element which undergoes *more* interactions than the wave does. This split again seems to be entirely arbitrary, so there really is no unified dynamics.

It's as if you forgot (didn't realize?) that in Bohm's theory the complete state description includes, in addition to the wf, the particle positions. Yes, in the 2 slit exp, there is nothing in the wave function from which you can infer precisely where a particle will end up... but the wave function isn't the only thing there. There is also *a particle* which follows some definite path and ends up at some particular spot. You don't have to magically collpase the wave function or anything like that to make the particle hit a particular spot on the screen.

It is sad that you are so vitriolic against Bohm, but don't even begin to understand how the theory works or why it is valuable (e.g., its solution to the m'ment prob).



:rolleyes: First of all, Vanesch would protest here. Second, most people do see that Bohmian mechanics gives the problem a more reasonable face, but since no real solution is presented and the conflict with GR and SR is blatant, most see it as window dressing.

Well that gets us back to the original point of this thread: NO THEORY CAN DO WITHOUT NONLOCALITY. If that's what you're saying bother you about Bohmian Mechanics (and that is obviously the source of its conflict with relativity) then you're just out of luck, because you cannot have a local theory that is consistent with the QM predictions and the data as we know it at present.

Oh, right, you want to take advantage of the experimental efficiency loophole. OK, fine, I hope they figure out how to do a better experiment soon to close that (and the "locality loophole" at the same time).


No, you still do not understand what I say (although Farsight seems to) : what you say is moreover trivial and can be found in any textbook. The problem is not the fact that I can see the wavefunction in two ways (that is just cheap blablaba), the issue is that it is very hard to believe that this non-local quantity which contains as well information about the future and the past can serve as a physical guidance mechanism; without people giving a more in depth mechanism which is closer to GR as why this should be so.

It's not clear what you mean by "closer to GR". If you mean, when it comes down to it, a more in depth mechanism "that isn't nonlocal" then you're out of luck. If you just want to embed Bohm's theory in a GR-like spacetime structure, there's no problem... except that you need to add spacetime structure to GR (e.g., a dynamically preferred foliation) to define the nonlocal dynamics. But then, there is only *one* example of a semi-reasonable theory for which you *don't* need to do this (namely Roderich Tumulka's version of GRW with Bell's "flash" ontology).
 
  • #182
ttn said:
Look, there are just two different possible attitudes you could take here. You could take the "completeness doctrine" at face value, and say that the wave function in OQM provides a literal description of the physical state of quantum systems.

Maybe you (or I) misunderstood the "completeness doctrine" then. I thought that Bohr meant to say: OQM does NOT provide a literal description for the physical state of a quantum system, AND NO SUCH STATE EXISTS. In other words, there IS no quantum ontology that is somehow modellisable by a mathematical construction. The only thing (for Bohr) that is real are events in a classical world, and these events just "happen", and he calls them measurements.
But, funnily, we have an algorithm to say something about their probabilities of happening. But there's nothing really going on beneath it. There IS no ontological quantum world.

Let me make it clear: I find that a difficult-to-accept attitude, but it lives on. It would be the ultimate demise of physics as an attempt to understand the world, because it is not describing it, it is only giving you some calculational tricks for "things that happen".

It is not entirely schizophrenic (only half way :smile: ). After all, Einstein once said: "the most incomprehensible thing about nature is that it is comprehensible". Well, Bohr answers: "well, it isn't comprehensible, after all". Things happen. But, we can estimate ourselves lucky to at least have found some statistical account of the things that happen.

Or, as you suggest, one could just forget about objective reality and use the QM formalism as a black box algorithm. But that is just failing to address the question at hand (about local causality), not answering it in a certain way (i.e., proving an example of a causally local theory, or proving that Bell Locality doesn't make sense or something).

Nevertheless, that's the spirit of OQM.

I mean, maybe we need to go back to the beginning. There *is* an objective reality, right?

Not according to OQM. Well, not on the quantum level according to OQM. Bohr's doctrine sees the strictly classical world as objectively real, but in which some "events happen". So the classical kinematics is true, but not classical dynamics, because it is perturbed by "things that happen" which are spurious "quantum processes", or "acts of measurement".
But there's no further reality underlying these processes - at least that's the way I understand his completeness doctrine. And I don't like it.

You're just not *talking* about it. But the question of whether or not the causality out there in the world is or isn't relativistically local, remains. Your not talking about objective reality right now doesn't make that question magically disappear or become meaningless.

Well, unless there IS no such objective world out there on the quantum level, and "things just happen". That's all what's in the word "realist", no ? The fact that there is a mathematically modelisable objective reality in the first place. Exactly the kind of thing that Bohr rejects.

So you just accept as an a priori truth that objective reality is deterministic. OK, I mean, I actually lean that way too. I wouldn't claim it as an a priori truth, but certainly all other things being equal it's better to have a deterministic theory than not -- especially since you could never possibly have a strong argument for the stochasticity in a given theory being irreducible (Patrick's theorem).

We're on exactly the same wavelength here. I have to admit that I feel extremely bad about Bohr's viewpoint - although it cannot be dismissed, but one should try to avoid it. And I'm also of the opinion that IF there is to be a mathematical description of an ontological reality (which Bohr rejects), then it must take on the form of a deterministic theory --- in which, of course, random variables might occur - which are variables to which one could assign a definite ontological value, but of which we ignore it, so we give it a probability distribution, which is our "ignorance description".

But, nevertheless, as a strategic point, I think it is very important to point out that Bell's inequalities in principle apply both to local deterministic and to local stochastic theories. You don't want to even consider the latter.

No, and you still do not get my point. I'm claiming that irreducibly stochastic theories ALWAYS have some "Bohr doctrine" to them, for which there is no modellisable ontological reality in the first place. Exactly as OQM - otherwise they would take on the form of a deterministic theory with random variables.
Once you drop the requirement of having an ontological reality which is mathematically describable, there's no such concept as causality anymore, because "things just happen" and all we have, at best, are statistical rules of these happenings. This can still be relativistically invariant, and in this case, Bell's locality condition becomes less evident, because there IS no causality in this setup, just rules of "things that happen" without any underlying ontology (apart from the "things that happen": a loose set of events, observations, if you want).

This is why I find the description of "local realist" rather accurate for Bell's condition: one can drop the "realist" part (namely, what you take as an a priori, that there IS a describable ontology with causal links) and just consider the "local" part as the relativistically invariant statistical rules that govern the "bag of events that just happen".

Just to make clear of what is the "ontological model" in this case:
{ (x1,y1,z1,t1: click) ; (x2,y2,z2,t2: push button) ; (x3,y3,z3,t3: reading = 5 V); (x4,y4,z4,t4: red light on) ; ...}: a simple list of events filling up the universe. No causal structure. Just events. And all things about "particles", "fields", ... are nothing else but algorithmic constructs to allow us to express observed statistical regularities in this list. This is how I see Bohr's doctrine.

Ok, fine, but some other people do, and it's important for them to know that they're barking up the wrong tree. If you're made uncomfortable by the non-locality Bell's Theorem proves must be present in any deterministic theory, then you should be *very* uncomfortable, because you CANNOT RESTORE LOCALITY BY DROPPING DETERMINISM.

But you might (although it is another form of locality) by dropping the attempt at constructing an ontological, causal model in the first place, and limit yourself to a set of statistical rules that events have to obey (without any claim of causality). If this set of statistical rules is relativistically invariant, then that's still "local" ; and then, signal locality is enough.

And that is true whether or not your philosophical sensibilities permit you to take irreducible stochasticity seriously.

I can take it seriously, but - as you do - I don't feel right with it. I prefer (as you do) an ontological model of reality, which, I think, should always be cast in the form of a deterministic theory (eventually with random variables describing our ignorance).

BTW, Patrick, does this mean you are unwilling to consider the GRW theory as a serious version of QM?

I don't know. GRW has the same problem as Bohm in that it is not relativistically invariant, and unless one finds a good explanation for relativity starting from other principles, I don't want to drop relativity.
GRW does something "nice" to OQM, which is to make appear in a natural way what is "an macroscopic" and what is a "microscopic and hence quantum" system. Now, I don't know too much about GRW, so I should be careful with my comments about it, but it seems to me as "fudge factor" kind of work. A physically based GRW, such as Penrose's approach, looks however more promising. If, at the same time, it can EXPLAIN why the world looks relativistic, then that would be a big leap forwards.

How about orthodox QM with a "cut" put in at some artibtrary level of "macroscopicness" (however that is measured)?

This is the other (and even more important) problem that I see with OQM: the arbitrariness in the Heisenberg cut.

I'm sorry, this just doesn't make any sense. "Signal locality" is about whether you can transmit a message faster than light. Bell Locality is about whether there are FTL causal influences. They're not just different "formulations" of the same concept, locality. They're about two very different things. So it's not an issue of "signal locality will do". If what you're interested in is whether it's possible to send signals, then yeah, signal locality will do. If, alternatively, what you're interested in is whether or not there exist FTL causal influences out there in the world, then only Bell Locality will do.

Exactly. That's the whole point. Now, if all you think, exists for real, are events, with NO causal influence, but just obeying some statistical distribution, then you see that signal locality is good enough.
If you think that there is causal influence, then one should go to Bell locality.

Let's not forget that the only reason to require locality is to keep relativity alive, not just "effective relativity" (as in an aether theory), but the principle of relativity. If there ARE causal influences, then the only way to describe these causal influences is by obeying Bell locality. True. If causality is not something that exists, and we just have a list of events which obeys certain statistical rules (~ Bohr's doctrine), then all relativity will require is that these statistical rules are invariant under Lorentz transformations. And then, signal locality is good enough.

Yes, they make the same predictions. But on the other hand THEY ARE COMPLETELY DIFFERENT THEORIES because they posit completely different ontologies.

No, that's what you don't seem to understand: OQM does NOT posit any ontology: it says that there IS no such thing as a quantum ontology.
(ok, it posits a classical ontology on the macroscopic level and denies an ontology on the microscopic level).

That's not quite phrased right. Bohmians think that Bohm's theory provides the best available candidate picture of the world. That picture is the ontology of the theory, just like some other picture is the ontology of MWI or of GRW. What is the confusion here?


"Bell locality of the predicted probabilities"? Sheesh. I can only correct your refusal to understand the meaning of "Bell Locality" so many times...

Yes, yes, I know. I know that "Bell locality" is a litmus test on a theory. But I losely use the word "Bell locality" for probabilities (whether observed or predicted) for sets of probabilities satifying all the possible Bell inequalities.

You're drunk or something. They're the *same* algorithm to calculate probabilites. Where they differ is in the ontology they posit (well, and the clarity of their formulations). You're now saying that really we shouldn't take the ontology of Bohm's theory seriously, and we should just consider it as another black box algorithm... except it's really just *the same* black box algorithm?? Did you overdose on some kind of positivism pills or something?

A misunderstanding here: OQM is an algorithm to predict probabilities, because for OQM THERE DOES NOT EXIST AN ONTOLOGY apart from a list of "events happening" (and hence no "causal" influences or what ever).
In the same way as you cannot compare apples to oranges, the only point of comparison of OQM (= an algorithm to predict probabilities) and Bohmian mechanics is, well, the predicted probabilities. They are the same.
But, as we've seen, OQM not positing any ontology, there's no causality either, so there's no application of Bell's Locality criterium, because there are no beable in OQM.
As I've said, for a probability algorithm, in order to be relativitically invariant, it is good enough to be signal local. OQM and Bohm, seen as a probability algorithm, both being identical, pass the test.
Now, Bohm also pretends to be an ontological description (while OQM tells you that there is no such thing). As such, one can apply Bell's locality criterium to it, and it fails. We also know, from the Bell inequalities, that any attempt at an ontological description of the predictions of OQM, will fail too.

Forget about "opening the box" of a theory. Start with the existence of a real world out there. Insist that there aren't any FTL causal influences.

But that is exactly the assumption that Bohr rejects.

Derive an inequality from this. Test this empirically and find that it's violated. Infer that there *do* exist nonlocal causal influences in nature.

Or, that there is no such thing as "causal influences" and that "things just happen" and that we find statistical descriptions of "what happens". That's Bohr's point of view.

The point is just the reverse - you infer that any theory at all is going to have to have nonlocal mechanisms "in its box", because we already know going in that NATURE is nonlocal.

No, because of two reasons. The first is Bohr's viewpoint: it can be that there ARE no causal influences, and that "things just happen" and all we can do is infer statistical regularities of the things that happen. As such, Bell's condition fails, because of the assumption of existing causality, which is not true here. As I said, I don't like that, but it is a possibility.

The other possibility is the tacit assumption that observations are unique events that happened, even though we can only know about it by later observation (upon meeting the other observer). As such, we have to take the word of the other observer that he saw something (fine) AND ALSO THE ASSUMPTION that there was no possible alternative other observer.
You know that this is the way out by MWI, which, in this way, is entirely Bell local, ontological-realist and even deterministic (but deluded).

So I claim that you make about the same error as Bohmians had to suffer from the rest of the community for years: the claim of impossibility while there exists an obvious counter example.
Bohm's theory suffered from it by von Neumann's erroneous theorem (a realist theory could not explain QM) which claimed it was impossible to build a theory as Bohmian mechanics ; while MWI suffers from an erroneous interpretation of Bell's theorem that any underlying realistic ontology must be non-local, MWI clearly being a counter example.
 
  • #183
Careful said:
My statement of course depends upon what you mean with the measurement problem. If you simply mean that you want to avoid the nonlocal collapse of the QM wave, then of course BM can do that - although it distinguishes itself then from standard QM in a measurable way. What this proposal does *not* adress is that upon a *non-physical* act of the observer (ah yes, no causal effects in the past lightcone here, no pre-determinism) a new local wave package (guiding wave) in position space is constructed.

You know, I thought that too, before. But (thanks to endless discussions with ttn here), I learned a bit more about BM, and there's no collapse at all in BM: you can have it evolve under the strict schroedinger equation (exactly as in MWI). If you do the math, you'll see that the "other branches" do not contribute anymore, for exactly the same reasons as decoherence in MWI.

(a) basically what is measurement ? (what do we call measurement of position of electron in atom)
(b) in a multiparticle system, God plays dice in configuration space. How can you have any hope of doing physics in this way ?
(c) Truely speaking, I cannot make sense out of non-local guidance mechanisms; what does it mean that our ignorance influences the dynamics of particles ?

You should read the paper ttn quoted.
(a) a measurement is the precise knowledge of all the particle positions in a certain subsystem (called measurement apparatus, or observer, if you want), after that this subsystem interacted (in any way you want) with another subsystem, the system under study. Given the non-local nature of the guiding wave forces, you can at most have a *correlation* between both systems, and it is shown that, given the square of the reduced wavefunction of the system, there's no correlation left between the particle positions in the system and in the observer. As such, most you can "extract" as information is the square of the wavefunction of the system under study. I have to say I was blown away by the argument (didn't study it in detail yet).

(d) in QFT on curved spacetime, the foliation would determine the choice of trajectories, what is the physical meaning of all this ?

Bohmian mechanics being non-local, it rejects of course relativity.

Actually Patrick, you keep on mentioning decoherence all the time, but don't you see that this is at least as conspirational as super-determinism ?? Take your decoherence argument to the Planck scale and try to figure out why long range entanglement (or even elementary particles) should exist !

I don't follow you here. Decoherence means simply that you CANNOT GET RID of entanglement. The more, the merrier. So giving more degrees of freedom, it will entangle even more.
Entanglement means absense of observed correlation (= interference).
That can sound bizarre of course: are the funny EPR correlations not DUE to entanglement ? In fact, no: they are due to the fact that we can entangle TWO systems (the two photons) and *protect* this entanglement from any further entanglement with the environment until spacelike observation. That's why it is hard to do with apples, and we need photons.
 
  • #184
** It's as if you forgot (didn't realize?) that in Bohm's theory the complete state description includes, in addition to the wf, the particle positions. Yes, in the 2 slit exp, there is nothing in the wave function from which you can infer precisely where a particle will end up... but the wave function isn't the only thing there. There is also *a particle* which follows some definite path and ends up at some particular spot. You don't have to magically collpase the wave function or anything like that to make the particle hit a particular spot on the screen. **

:grumpy: No, the sad thing is that you do not understand that *even* if the particle follows a deterministic path, to make it VISIBLE/OBSERVABLE you need to let it interact with other *classical* fields the wave does not interact with. There is still an ad hoc split quantum/classical in your thinking.

**
If that's what you're saying bother you about Bohmian Mechanics (and that is obviously the source of its conflict with relativity) then you're just out of luck, because you cannot have a local theory that is consistent with the QM predictions and the data as we know it at present. **

Sure we can have, but not in the way Bell saw it. Basically, either you don't understand the ways you can go beyond Bell or you are manifestly cheating. That is all there is to it, so your comment :

**
Oh, right, you want to take advantage of the experimental efficiency loophole. OK, fine, I hope they figure out how to do a better experiment soon to close that (and the "locality loophole" at the same time). **

indicates either one of these.

I have clearly explained two local mechanisms based upon correlations Bell threw out of the window, suitable for doing this. Perhaps you should tell us who you are, that we can read one of your papers - my identity has been given away here for free already.

Careful
 
Last edited:
  • #185
**
You should read the paper ttn quoted.
(a) a measurement is the precise knowledge of all the particle positions in a certain subsystem (called measurement apparatus, or observer, if you want), after that this subsystem interacted (in any way you want) with another subsystem, the system under study. Given the non-local nature of the guiding wave forces, you can at most have a *correlation* between both systems, and it is shown that, given the square of the reduced wavefunction of the system, there's no correlation left between the particle positions in the system and in the observer. As such, most you can "extract" as information is the square of the wavefunction of the system under study. I have to say I was blown away by the argument (didn't study it in detail yet). **

But the point is that you *cannot* know the exact particle positions in **reality**, it is merely an artificial construct to promote these things as classical *observables*. But I shall read the paper.
 
Last edited:
  • #186
Careful said:
:grumpy: No, the sad thing is that you do not understand that *even* if the particle follows a deterministic path, to make it VISIBLE/OBSERVABLE you need to let it interact with other *classical* fields the wave does not interact with. There is still an ad hoc split quantum/classical in your thinking.

Um, actually it's you who splits quantum from classical. I'm the one who is a fan of Bohm's theory, in which there simply is no such distinction. That's one of the beautiful things about Bohm's theory -- there aren't two separate realms/worlds (quantum and classical) which obey different dynamical laws, leaving us with the problem of determining how they interact, what rules govern that interaction, where exactly the "cut" between them is, etc. No, Bohm's theory is a theory about a single whole universe made of particles moving under the guidance of the wf. That universe includes assemblages of particles that are tables, chairs, particle detectors, cats, dogs, people, etc.

It's only from an orthodox/dualist point of view that you'd think that something special needs to happen for some *real quantum fact* to become observable or whatever. And that's because in muddled orthodox thinking, nobody's ever quite sure if "real" really means real, or whether there's really a quantum world at all, or just classical, etc. And this is exactly what's leading you to insist that something more is needed, even after (acc to Bohm's theory) particles, which really exist, end up in some place.

Maybe it would help to make the example a bit more crude. Suppose I set up a rube goldberg device which moves my dining room table into the living room if a certain spin 1/2 particle comes out the "up" port of a S-G device, and it moves my dining room table into the bedroom if the particle comes out "down." Such a process can be described completely within Bohmian mechanics, without ever setting the "quantum" dynamics aside and "collapsing the wave function" or "bringing in a classical observer" or anything like that. There's just some fact of the matter about where all those particles (which constitute the table) end up -- living room or bedroom.

And this, by the way, is exactly why there is NO MEASUREMENT PROBLEM in Bohmian Mechanics.


I have clearly explained two local mechanisms based upon correlations Bell threw out of the window, suitable for doing this. Perhaps you should tell us who you are, that we can read one of your papers - my identity has been given away here for free already.

So has mine, though I would prefer to remain anonymous, frankly. But if you want to get personal, just send me a PM and I'll send you a private email.
 
  • #187
Sorry hurkyl, I don't mean to be dogmatic.

Interesting stuff all this guys. I'm learning something.

http://64.233.183.104/search?q=cache:J-nWSzlSt2QJ:www.citebase.org/cgi-bin/fulltext%3Fformat%3Dapplication/pdf%26identifier%3Doai:arXiv.org:quant-ph/0603027+%22gwr+theory%22&hl=en&gl=uk&ct=clnk&cd=6 [Broken]
 
Last edited by a moderator:
  • #188
Careful said:
But the point is that you *cannot* know the exact particle positions in **reality**, it is merely an artificial construct to promote these things as classical *observables*. But I shall read the paper.

Eh, I don't see your point. When looking at a theory, I always try to take the position: "let's see what this theory says in a toy universe where it holds strictly". This is BTW, how I arrive at MWI for QM.
This doesn't mean that I think that said theory corresponds to our universe! I just say: if I were a god, and I made a universe according to theory X, where I decide it to hold strictly (it's My Law, after all!), what kind of universe do we get ?

Now, in Bohmian mechanics, there is a fixed, Newtonian background space, and an absolute time, so relativity is out of the window ; and there are particles, and a wavefunction. Observers are made of particles, so their memory is made of particles, but there is also this "ghost field" in configuration space, which is the wavefunction. It evolves strictly according to the schroedinger equation, no collapse, no "special observer" no nothing.

Measurements are nothing else but the act of putting data in a memory, which is made up of particles, about other particles' position.
And then it turns out, according to the fixed, deterministic dynamics of BM, that, assuming some initial "statistical mechanics" hypothesis, that if you repeat enough a similar experiment, that the data you will collect, in your memory, about other particles' position, is distributed according to the reduced wavefunction of those other particles. It *follows* from the mechanics of BM, it isn't put in.
As such, entire equivalence of observation with standard QM is obtained. Well, in fact, more with MWI, in the sense that "quantum interference" is never irreversibly removed, as it is in standard QM, because in the wavefunction, all "other branches" still live on, but have, in most cases, no genuine contribution anymore, exactly as with decoherence.

So, I concur with ttn, that there is no measurement problem in BM. In fact, I thought for a long time that there still was, because of a mixture of objective states and the need for a very special initial "quantum equilibrium condition" which is needed to make BM coincide with QM. But the paper that ttn quoted seems to indicate that you can make just about any initial distribution. For most typical distributions (not necessarily the quantum equilibrium condition), you'll end up with an effective quantum equilibrium condition if there's enough stuff whirling around. A bit as with thermodynamics.

In this sense, BM is far superior over QM.
So why am I not a Bohmian ? Because in this toy universe, relativity is out of the window. That's all.
 
Last edited:
  • #189
vanesch said:
So, I concur with ttn, that there is no measurement problem in BM. In fact, I thought for a long time that there still was, because of a mixture of objective states and the need for a very special initial "quantum equilibrium condition" which is needed to make BM coincide with QM. But the paper that ttn quoted seems to indicate that you can make just about any initial distribution. For most typical distributions (not necessarily the quantum equilibrium condition), you'll end up with an effective quantum equilibrium condition if there's enough stuff whirling around. A bit as with thermodynamics.

Yes, good. For the benefit of lurkers, there is a lot of interesting stuff brought in with the word "typical". To say something like "for most possible initial conditions, you get the usual QM predictions" you need some way of defining "most" -- which means, some way of counting all the possible initial conditions. Of course, since what we're talking about is the initial positions of particles -- continuous variables -- there's an infinite number of possible initial conditions. So you need some kind of *measure* over them. But how to choose this? The whole thing depends on it. For some measures, the claim in question ("for most possible initial conditions, you get the usual QM predictions") will be true, and for some it will be false. The argument in the paper I cited is that the already-specified dynamical laws of Bohmian Mechanics point to a certain dynamically-preferred measure -- namely, the one which is (so to speak) preserved by time-evolution in the theory. And then (hooray!) relative to this dynamically preferred measure, it is true that *typical* initial conditions result in familiar QM predictions. And, as Patrick said, this is indeed very very much like ordinary statistical mechanics. You don't need to go in by hand and pick out some ridiculously special initial conditions to "fine tune" things to make the theory work out. No, you can just imagine God throwing the dice once at the beginning of the universe (funny shaped dice, though) and then you're overwhelmingly likely to end up with a world that pretty much looks like our world. Just like in classical stat mech... in which there *are* initial conditions that would result in coffee cup parts jumping spontaneously from the floor up onto the table and reassembling themselves (and so forth). It's just that such initial conditions are exceeding special/rare, and so we can rest satisfied with an explanation which posits some genuinely random initial conditions in the past.


Patrick, I'm not going to bother to reply to your last long email about Bell Locality and surrounding issues. I think we've reached the usual point of going around in circles without either of us moving. I disagree with much of what you said, but I think I've already expressed my disagreement over those points, so it would just be pointlessly repetitive to, uh, repeat them.

Plus, now that you're defending Bohm's theory (against goofy misunderstandings) I don't want to induce any perturbations in the system. :tongue2:


In this sense, BM is far superior over QM.

I think, for anyone who really understands both theories, there can be no doubt about that.


So why am I not a Bohmian ? Because in this toy universe, relativity is out of the window. That's all.

Even though I think it's dead wrong, this is at least a sensible position. BTW, the main reason it's dead wrong is that relativity is also out the window with MWI, and in several senses. First off, it's difficult if not impossible to find any ordinary 4-D relativistic material world in MWI. Whatever hints of relativity there *are* to be found in MWI, are very abstract -- like the laws of evolution for the MWI-universe are Lorentz invariant or whatever. But that is just an empty, formal shell of what relativity is really all about -- which is the spacetime structure of the world. If MWI doesn't even have a spacetime in it, how can you take seriously the idea that it's "relativistic"? Second, in MWI we have to accept that ordinary sense perception is delusional. What we see (e.g., cats that are definitely alive or dead, tables that are here and not there, etc.) is simpy *not* the way things really are. (Just as, in the context of OQM, say for a particle whose wf is split 50/50 between two distinct spatial regions (L and R), if somebody said "REALLY, the particle is over there on the L" he would simply be *wrong*. If the particle's wf is a superposition of L and R, then it is *false* that the particle is definitely on the L.) And once you reject the idea of direct perceptual access to reality (i.e., say that perceptual experience is systematically deluding you) then you can forget about EMPIRICISM -- you can forget about the idea of trying to base physical theories on the results of *experiments*. (Why? because when you do an experiment and try to learn the result by looking at the apparatus pointer afterwards, you don't learn it! you get deluded!) And this includes all the experiments that made people believe in relativity in the first place. So, in short, according to MWI, there can be no empirical/scientific reason to believe in relativity -- which makes for a big logical circle, since it was only to allegedly *save* relativity (from Bell's onslaught) that we were supposed to consider something as crazy as MWI in the first place.
 
  • #190
**
So has mine, though I would prefer to remain anonymous, frankly. But if you want to get personal, just send me a PM and I'll send you a private email. **

Well, I prefer too but sometimes people just bend rules the way they want to. The PM is a done deal ... you should have it now. Let's discuss privately further, I think the issues concerning Bell, GR incompatibility, and lack of understanding of guidance wave (from a sensible spacetime perspective) are done deals (although I would have appreciated a more balanced answer from your side). I understand the sense in which you believe it solves the measurement problem (there is still the micro macro one), but let's keep that for PM.

Careful.
 
  • #191
ttn said:
Even though I think it's dead wrong, this is at least a sensible position. BTW, the main reason it's dead wrong is that relativity is also out the window with MWI, and in several senses. First off, it's difficult if not impossible to find any ordinary 4-D relativistic material world in MWI.

First of all, let's put straight what I mean with "relativistic" here.
I mean: special relativity (Minkowski space) OR, "kinematical" general relativity (that is, the geometry is GIVEN).

Of course I do not mean any quantum theory of gravity. Face it, we don't have any. So I'm talking about a world where there is no active dynamical gravity. This is the toy universe where I consider MWI to be valid, and this is the toy universe which I propose for the interpretation of QM.
I'm not sure, nor even convinced, that quantum theory will survive in its present form in any theory of gravity.

Whatever hints of relativity there *are* to be found in MWI, are very abstract -- like the laws of evolution for the MWI-universe are Lorentz invariant or whatever. But that is just an empty, formal shell of what relativity is really all about -- which is the spacetime structure of the world.

I've said this already before: you think of course that I think that the WAVEFUNCTION is "ontologically real". No, it is the entire unitary structure, which, in a specific coordinate system splits in a wavefunction, and a time evolution operator. These transform under a representation of the Lorentz transform (just as spacetime coordinates transform under a lorentz transformation, and the manifold of the spacetime points is the "ontological structure").

If MWI doesn't even have a spacetime in it, how can you take seriously the idea that it's "relativistic"?

MWI has in it that its "ontological structure" is a unitary structure which obeys symmetries given by a representation of the Lorentz group. So the "beables" (of which a "coordinate representation" is a split in a wavefunction and a time evolution operator) are relativistically invariant.

Second, in MWI we have to accept that ordinary sense perception is delusional. What we see (e.g., cats that are definitely alive or dead, tables that are here and not there, etc.) is simpy *not* the way things really are.

Yes, but they are not totally wrong either. You only see ONE ASPECT of a more complete reality. That doesn't mean that what you see is wrong, what is wrong is just to assume that what you see is ALL there is.
That's not completely delusional. It is half delusional. We've won 50%.

(Just as, in the context of OQM, say for a particle whose wf is split 50/50 between two distinct spatial regions (L and R), if somebody said "REALLY, the particle is over there on the L" he would simply be *wrong*. If the particle's wf is a superposition of L and R, then it is *false* that the particle is definitely on the L.)

Well, no. If somebody said: the particle is on the L, he would not be wrong. He would simply not be complete, because the particle is ALSO on the R. He would be wrong if he said: the particle is ONLY on L.

And once you reject the idea of direct perceptual access to reality (i.e., say that perceptual experience is systematically deluding you) then you can forget about EMPIRICISM -- you can forget about the idea of trying to base physical theories on the results of *experiments*. (Why? because when you do an experiment and try to learn the result by looking at the apparatus pointer afterwards, you don't learn it! you get deluded!) And this includes all the experiments that made people believe in relativity in the first place.

No, that's not true. You can only conclude that your observations are only part of the story. And by trying to think about it, to complete the story. It is not because you only see the skin of an apple that there is no interior, right ? But you can never see the interior of objects, only their surface (that's a metaphor). But it is not because you only see surfaces, that you cannot DEDUCE that there are interiors.
It is only in rare events that you get an indication of an access to the interior: when taking, say, X-rays. Well, that's like quantum experiments. You get results that make you sometimes conclude that the particle was at L AND ALSO AT R. You're not used to thinking that way, but it can be done. And then you say: why doesn't this apply to other observations ?
And hence deduce an MWI kind of picture.

So, in short, according to MWI, there can be no empirical/scientific reason to believe in relativity -- which makes for a big logical circle, since it was only to allegedly *save* relativity (from Bell's onslaught) that we were supposed to consider something as crazy as MWI in the first place.

No, again, because we're not totally deluded, we only see *part* of reality, and hence complete it by certain indications, and further thinking. Just as our hypothesis of the interior of an apple is.

And, as I said, MWI is only an interpretation in a toy universe where QM holds rigorously. Which it probably doesn't in ours - but then we don't know. Maybe MWI can be extended to a "quantum theory of gravity", or maybe not.

You think of course that you already have a "quantum theory of gravity"...
BTW, question: in Bohmian mechanics, is there Hawking radiation from a black hole, or not ?
 
  • #192
vanesch said:
First of all, let's put straight what I mean with "relativistic" here.
I mean: special relativity (Minkowski space) OR, "kinematical" general relativity (that is, the geometry is GIVEN).

Of course I do not mean any quantum theory of gravity. Face it, we don't have any. So I'm talking about a world where there is no active dynamical gravity. This is the toy universe where I consider MWI to be valid, and this is the toy universe which I propose for the interpretation of QM.
I'm not sure, nor even convinced, that quantum theory will survive in its present form in any theory of gravity.

Yes, fine, that's the normal context I take for granted as well.



I've said this already before: you think of course that I think that the WAVEFUNCTION is "ontologically real". No, it is the entire unitary structure, which, in a specific coordinate system splits in a wavefunction, and a time evolution operator. These transform under a representation of the Lorentz transform (just as spacetime coordinates transform under a lorentz transformation, and the manifold of the spacetime points is the "ontological structure").

I don't know why you'd want to include the time-evolution operator in the ontology of the theory. Isn't that like saying, according to classical physics, "the world is built out of massive particles and 'F=ma'"? But that makes no sense. "F=ma" is not a separate, additional thing that exists. It's just a description of the actions of whatever does exist -- particles and such in classical physics. And I think it's just parallel for MWI: the wf provides the complete ontology, and then there is some analogue of "F=ma" that describes the time evolution of the state. I don't think anything we've been arguing about hinges on this, though, so maybe it's not worth pursuing.


Yes, but they are not totally wrong either. You only see ONE ASPECT of a more complete reality. That doesn't mean that what you see is wrong, what is wrong is just to assume that what you see is ALL there is. That's not completely delusional. It is half delusional. We've won 50%.

I'm sorry, but the truth status of propositions is like pregnancy. You can't be half pregnant, and a proposition can't be half true. If a proposition captures a literal *part* of reality, then it is not "partly true", it is just plain true. For example: "there's a cat sleeping on my bed" is 100% true, even though it obviously fails to state lots of other things that are also simultaneously true. On the other hand, if you state something which is in fact false, then it is just plain false -- not "half true" -- even if there is some way of twisting words to make it feel like, in some weird sense, what you say caputres a "part" of reality. Unfortunately, this point hinges on getting clear about the meaning of certain words like "part" and "and", so it's hard to make headway on. But let me put it this way. Superposition, in ordinary quantum mechanics, is different from the normal "and". If a particle is in a superposition of being located on the L and being located on the R, it is often OK in a loose way of speaking to say "it is on the L *and* it is on the R". And if you take that loose talk too seriously, I can see how you might erroneously conclude that the statement "it is on the L" captures "part" of the whole reality and is hence true. If "A and B" is true, then "A" is true, right? That's your argument. But my point is that it is too loose -- indeed, just plain wrong -- to translate the idea of superposition into the word "and" so you can play this logical game. When a particle is in a superposition of being on the L and being on the R, it is *not* on the L, it is *not* on the R, it is *not* both on the L and the R, and it is *not* neither on the L or the R. It's just not any of those normal classical things that we're used to. (Here I'm taking for granted an orthodox view in which the wf provides already a complete description, and there is just nothing for the word "particle" to refer to but the wave function.) So then your inference from "A and B" to "A" fails. Or rather, the translation of "(A + B)/sqrt(2)" into "A and B" is shown to be totally invalid/misleading/unwarranted. Does "A" follow (as a truth) from the assumed truth of "(A + B)/sqrt(2)"? I say: no way. To say that it does (or even that it could) is to completely undermine the assumption we've started from -- that the wave function alone provides a complete description of the facts. It would be to adopt an "ignorance interpretation" of superpositions -- i.e., to reject the completeness doctrine and endorse a naive hidden variable attitude... as in: what "(A+B)/sqrt(2)" *means* is just "either A or B, but nobody's sure which". And that is decidedly *not* the orthodox view that MWI asks us to accept. MWI is *not* a theory that says: there's a big complicated wave function with lots of branches, and one of those describes the actual state of the world -- we're just not sure which. But it is only in such a theory (the one that is decidedly *not* MWI) that it would make sense to say that when the wf is "(A+B)/sqrt(2)", a belief such as "A" might be true. And since MWI is not this theory, we cannot say this in MWI -- i.e., we must admit that all of our perceptual judgments are false; perception is delusional. Bye-bye empiricism.



Well, no. If somebody said: the particle is on the L, he would not be wrong. He would simply not be complete, because the particle is ALSO on the R. He would be wrong if he said: the particle is ONLY on L.

As I explained above, I don't think it makes sense to say "the particle is ALSO on the R." That's just not what superpositions are supposed to mean in OQM or MWI.

But your last sentence makes me think that argument might not even be necessary. When a person says "the particle is on the L", doesn't he *mean* (perhaps by the tacit implications of the concept "particle") that "the particle is ONLY on the L"? I mean maybe then you'd want to say the guy's problem isn't with L/R but with his concept of "particle" -- he thinks there's a thing there that *has* a definite location (a "particle") and really there is just no such thing. There's instead this kind of spread out thing. But that doesn't change my point at all. Either way, he's seriously wrong about what is really going on out there independent of him. His beliefs -- which in this case are based on *direct perceptual experience* -- are false. His perception is delusional. And so bye-bye empiricism...



No, that's not true. You can only conclude that your observations are only part of the story. And by trying to think about it, to complete the story. It is not because you only see the skin of an apple that there is no interior, right ? But you can never see the interior of objects, only their surface (that's a metaphor). But it is not because you only see surfaces, that you cannot DEDUCE that there are interiors.

The problem with this metaphor is that it breaks down -- it just doesn't have the right kind of analogy to the case at hand. The skin/inside of an apple is analogous to the cat being on the bed and (say) the TV being on the table. These are two different facts pertaining to two different things -- two *parts* of the world in the literal/normal sense of "part". They are not two contradictory claims about the same one thing, which is what is going on with the superposed particle. Well, "contradictory" isn't exactly right, because we can accept superposition. But the point is, two states of the particle such as "L" and "(L+R)/sqrt(2)" are two *different* states *of the particle*. They can't both be true at the same time. So it'd be like saying "the cat is on the bed" is true in a situation where, in fact, the cat is outside on the deck. The cat being on the bed and the cat being on the deck, are just two distinct possibilities. They can't both be true at the same time. Likewise with "L" and "(L+R)/sqrt(2)" -- *according* to OQM/MWI which insist that the wf alone (which is what "L" etc. stand for here, definite wf's) provides a complete description.



You think of course that you already have a "quantum theory of gravity"...

I do?? I don't think I think that. But maybe this just proves that I'm deluded and that MWI is true after all.


BTW, question: in Bohmian mechanics, is there Hawking radiation from a black hole, or not ?

Bohmian Mechanics (strictly speaking) is a theory of N-particle non-relativistic situations. There are various possible/tentative/half-worked-out extensions to the relativistic realm (but even this is fraught with confusion because *some* of relativity has got to go out the window in these generalizations, and it is not at all clear which aspects need to be kept and which pitched). Much less for QFT on a non-flat spacetime background. Or at least, I don't know of any. So there's no way (for me, now) to answer this question. But why in the world did you ask it anyway?
 
  • #193
***
I don't follow you here. Decoherence means simply that you CANNOT GET RID of entanglement. The more, the merrier. So giving more degrees of freedom, it will entangle even more.
Entanglement means absense of observed correlation (= interference).
That can sound bizarre of course: are the funny EPR correlations not DUE to entanglement ? In fact, no: they are due to the fact that we can entangle TWO systems (the two photons) and *protect* this entanglement from any further entanglement with the environment until spacelike observation. That's why it is hard to do with apples, and we need photons.***


Exactly, why would entanglement not produce a world which is on macroscopic scales (say order above 10^(-6) metres) so crazy that it does not resemble our classical world in any respect ? Why would it be possible to protect entangled electrons anyway against decoherence through interaction with the local environments ? People often argue that somehow this entanglement with the environment averages out (so that it is unimportant); neither did I ever see a convincing demonstration of this, nor is it clear that this is somehow scale dependent ?

Now, the elementary particles are BIG objects with respect to Planck scale degrees of freedom, so why should nature keep these coherent when you claim it is unlikely for apples ?

This is the reason why I do not believe that BM does solve the measurement problem (although it is certainly superior to all other ansatze in the ``standard'' literature - except perhaps for GRW) : when you let the Schroedinger evolution run, then the guiding wave will get slowly more and more non-local and entangled. It might very well be that after a sufficiently long amount of time everything gets entangled with everything so that it will be impossible to deduce from correlations the local laws of physics, how to proceed in this way? Perhaps I see it too dark, I would certainly appreciate comments ...

Anyway, in QG you will have to wonder why there are scales with blatantly different phemenology.

Out of courtesy, I shall first discuss privatly the other issues with ttn.

Careful
 
Last edited:
  • #194
ttn said:
I don't know why you'd want to include the time-evolution operator in the ontology of the theory. Isn't that like saying, according to classical physics, "the world is built out of massive particles and 'F=ma'"? But that makes no sense. "F=ma" is not a separate, additional thing that exists. It's just a description of the actions of whatever does exist -- particles and such in classical physics.

Ah, I have another view on classical physics: to me, the classical physics ontology is a flow on a phase space. An experienced world is a point on this phase space, following the flow.
As I argued elsewhere, there might be multiple worlds in the same classical physics ontology (different points evolving in this phase space), but there's no way to know. Maybe even the congruence of solutions is the entire ontology and all worlds somehow "exist" (of which we experience one). But they never interact.

And I think it's just parallel for MWI: the wf provides the complete ontology, and then there is some analogue of "F=ma" that describes the time evolution of the state. I don't think anything we've been arguing about hinges on this, though, so maybe it's not worth pursuing.

Ok.

I'm sorry, but the truth status of propositions is like pregnancy. You can't be half pregnant, and a proposition can't be half true. If a proposition captures a literal *part* of reality, then it is not "partly true", it is just plain true.

Yes, it is just poorly expressed, that's all.

For example: "there's a cat sleeping on my bed" is 100% true, even though it obviously fails to state lots of other things that are also simultaneously true.

The statement should then be: "I observe a cat sleeping on my bed", or "the cat is in a superposition of me seeing her sleeping on my bed, and a copy of me seeing her catching some mice" or whatever.

On the other hand, if you state something which is in fact false, then it is just plain false -- not "half true" -- even if there is some way of twisting words to make it feel like, in some weird sense, what you say caputres a "part" of reality. Unfortunately, this point hinges on getting clear about the meaning of certain words like "part" and "and", so it's hard to make headway on. But let me put it this way. Superposition, in ordinary quantum mechanics, is different from the normal "and". If a particle is in a superposition of being located on the L and being located on the R, it is often OK in a loose way of speaking to say "it is on the L *and* it is on the R". And if you take that loose talk too seriously, I can see how you might erroneously conclude that the statement "it is on the L" captures "part" of the whole reality and is hence true. If "A and B" is true, then "A" is true, right? That's your argument. But my point is that it is too loose -- indeed, just plain wrong -- to translate the idea of superposition into the word "and" so you can play this logical game. When a particle is in a superposition of being on the L and being on the R, it is *not* on the L, it is *not* on the R, it is *not* both on the L and the R, and it is *not* neither on the L or the R. It's just not any of those normal classical things that we're used to.

What's wrong with saying that it is in L and in R with different amplitudes ?

Like, what's wrong with saying that the light pulse is both a bit on L and on R ?

So then your inference from "A and B" to "A" fails. Or rather, the translation of "(A + B)/sqrt(2)" into "A and B" is shown to be totally invalid/misleading/unwarranted. Does "A" follow (as a truth) from the assumed truth of "(A + B)/sqrt(2)"? I say: no way. To say that it does (or even that it could) is to completely undermine the assumption we've started from -- that the wave function alone provides a complete description of the facts. It would be to adopt an "ignorance interpretation" of superpositions -- i.e., to reject the completeness doctrine and endorse a naive hidden variable attitude... as in: what "(A+B)/sqrt(2)" *means* is just "either A or B, but nobody's sure which".

Nono, if you say, it is BOTH at L and R, you don't mean, it is OR at L or at R, but I don't know. That would be the ignorance interpretation. No, the electron is BOTH at L and at R. It went through both slits. The light pulse went to both slits, and gave an interference pattern.

So after the electron went through both slits, it now is everywhere on the screen, and I observe one position of it (while my copies observe other positions of it on the screen).

If I now say: "the electron hit at x = 0.5 cm", then this is a shortcut. I mean: I observed the electron hit at x = 0.5 cm, while my copies observed it at x = 0.4, x = 0.3 ... The electron really was at position x = 0.3 AND at position x = 0.4 AND at position x = 0.5, they were observed each by copies of myself, and "I" saw it at x = 0.5 cm.

MWI is *not* a theory that says: there's a big complicated wave function with lots of branches, and one of those describes the actual state of the world -- we're just not sure which. But it is only in such a theory (the one that is decidedly *not* MWI) that it would make sense to say that when the wf is "(A+B)/sqrt(2)", a belief such as "A" might be true. And since MWI is not this theory, we cannot say this in MWI -- i.e., we must admit that all of our perceptual judgments are false; perception is delusional. Bye-bye empiricism.

No, we only have to include our copies, which saw the other aspects of reality, that's all.

Again, by thinking, you can extrapolate from what you observe, to what might really be there. If you have a theory of an ontology, and a theory of how observations are deduced from it, then that is just as falsifiable as having a theory of what is an ontology, and assuming naive realism, that what you observe, is fully equal to all of the ontology.

As I explained above, I don't think it makes sense to say "the particle is ALSO on the R." That's just not what superpositions are supposed to mean in OQM or MWI.

In OQM, it means nothing. It is a calculational trick.
In MWI, that's nevertheless exactly what it means: that the particle is BOTH at L and at R.

Either way, he's seriously wrong about what is really going on out there independent of him. His beliefs -- which in this case are based on *direct perceptual experience* -- are false. His perception is delusional. And so bye-bye empiricism...

Only if you're naive enough to think that what you perceive is reality and all of reality of course. Why should such a thing be true ? It is sufficient to have an explanation of your perception - and finding this in agreement with what you actually observe - to find an argument in favor of a certain view.




But the point is, two states of the particle such as "L" and "(L+R)/sqrt(2)" are two *different* states *of the particle*. They can't both be true at the same time.

Well, that's EXACTLY the content of the superposition principle: if a thing can be in a state A and a state B, then it can be in both at once, with different complex coefficients, and these are new states of the thing.

Admittedly the weirdest proposal in all of physics. MWI is just the consequent application of that principle: if I observe a dead cat and I observe a live cat, then all complex combinations of both are also possible.

Now, if I take the superposition principle as a deep, fundamental principle of nature, then I'm not surprised to arrive at MWI happenings: it was build in from the start !

There are other strange principles: take the equivalence principle: apples don't fall downwards, we see them fall because the surface of the Earth is in fact accelerating upward.

Or: the speed of a lightbeam for two observers in relative motion is the same.


So it'd be like saying "the cat is on the bed" is true in a situation where, in fact, the cat is outside on the deck. The cat being on the bed and the cat being on the deck, are just two distinct possibilities. They can't both be true at the same time.

Nevertheless, that is exactly what the superposition says...

Your reasoning is a bit: the surface of the Earth cannot be accelerating upward, because it is a sphere of fixed diameter. But that is forgetting the fact that spacetime can be curved.

In the same way, things we thought could be only in "one state at once" can be in a superposition of these states, even states of knowledge such as "I know the cat is on my bed" and "I know the cat is chasing mice". And experiences: I can experience the cat on my bed, and at the same time, I can experience the cat chasing mice. But, because "I" is nothing else but an experience, this means this is then simply dedoubled.
We now even have the *explanation* of why we think that certain things (like cats being on beds) can only be in one state, and not in superpositions (while they have to, obeying the superposition principle): that is because our body "experience states" (which ARE us) simply get quickly entangled only with one of them.

As I said somewhere else, MWI is not some desperate attempt at erroneously interpreting what comes out of a mathematical formalism (wavefunctions and so on). It is the rigorous and ontological interpretation of a fundamental principle: the superposition principle.
It would even lead to an MWI view "without wavefunction": if the principle says that what can be A and what can be B, can also be any thinkable combination of it with complex weights, then you have to agree that if I (= my body) can be in the grocery store, and I can be lying on my bed, then I can be in any combination with complex weights of being both in the grocery store and lying on my bed. These, however, corresponding to two different experiences I can have, it simply means that these two experiences are there, and "I" am just one of them.

Weird, no ? But what I mean is: it follows *directly* from the very first, fundamental, founding postulate of quantum theory, even before I set up the formalism. So one shouldn't be surprised not to get around it when a mathematical formalism is build upon this principle.

The only thing that is not in this formalism, explicitly, is what aspect of reality corresponds to "experiences" (as were points in phase space in classical mechanics). We take it that it are specific bodystates which correspond to specific experiences. This is then the added "theory of conscious perception".
 
  • #195
vanesch said:
Well, that's EXACTLY the content of the superposition principle: if a thing can be in a state A and a state B, then it can be in both at once, with different complex coefficients, and these are new states of the thing.

You're making my point for me. A is a possible state; B is a possible state; (A+B)/sqrt(2) is a possible state. (and lots more) But the crucial point is that the third is not the same as the first or the second. There are just these three distinct states. And so if the real state is the third, and someone believes it's the first, they're wrong. That's my whole point.


In the same way, things we thought could be only in "one state at once" can be in a superposition of these states,

This is misleading. Things *can* only be in one state at once. What the superposition principle adds is... well... a whole bunch of new possible states for things to be in. If, classically, you thought there were only two possible states (say), then once you allow superpositions there are not just two, but an infinite number -- all possible linear combinations of those two (which now form merely a basis for a whole vector space of possible states). It's misleading to say that a superposition is "being in both states at once". This suggests that really there are still only the two states (and that the laws of logic just became fluid). The real truth is just that there are now lots of possible states. But logic is still fine: if the reality is one state, and somebody's belief corresponds to some other state, then the belief doesn't correspond to the reality -- i.e., it is false.
 
  • #196
All the arguments seem to be --- as I earlier remarked ---- descending to disagreements on terminology and in particular ontology. This, in the Oxford companion to Philosophy, is described as concerning (among other things) 'the nature of existence and the categorial structure of reality'. If that is true there cannot be a theory with no underlying ontology. Even the purported statement that 'there is no ontology' is itself ontological, since it categorises 'reality ' in denying it.

It is not that I don't understand the statements made in posts , but that I do.
To be brutal, much of the argument is like candyfloss--when you bite, there is very little there. I am sure that both Einstein and Feynman made statements to the effect that if you couldn't explain it to (here I paraphrase) an uneducated person there was something wrong.

Please come back to layman's language. It will help to clear the air. For example , so far as I know, no-one commented on the extract from conversations I had with Bell. I quote:

" 3. On at least one occasion it was agreed that we all believed statements like "The book in a drawer is still there even when not being observed" to be true. If some versions of physics did not agree, whether as accidental fact or by the definition of physics, too bad for those versions.
4. The experimental results disproved the EPR thesis, and implied superluminality."

Which bits of that do the contestants disagree with and why? Clearly a Copenhagen adherent ( a breed I thought extinct) would call the para3 meaningless. But that is dogma; not an argument.

Ernies
 
  • #197
Hi Ernies,

Yep, I perfectly agree with you, if some people believe their approach solves the measurement problem ... let it be.
To comment on your conversations, yes the book is there even if we do not observe it and yes superluminality is a possibility (although I would prefer to have some upperbound on the velocity V in a particular Lorentz frame to preserve locality in a reasonable way). In this way, it still would be possible to have perfect EPR detections on scales proportional to tV where t is a typical registration time (of the order of a nanosecond). Now - listen before you jump out of your chair - Bell's INEQUALITIES besides allowing (reasonably) for superdeterminism with the real stuff not traveling FTL, is NOT in conflict with the statistical interpretation of QM and Bell locality ! This is a point made by Barut long time ago, this works if you *assume* that some fundamental law forbids the existence of a perfect measurement apparatus (just as the second law of thermodynamics forbids a perpetuum mobile). Ordinary Copenhagen *only* says something about the statistics of the *measured pairs*, and as you know (perhaps) very well there exist already for 30 years Bell local theories of this type with a very high measurement efficiency treshold. It are however those interpretations/alternatives which say something about individual particle events which take the point of view Bell did (which even contradicts the assumptions of QM - check out Barut for that).

**
Please come back to layman's language. It will help to clear the air. For example , so far as I know, no-one commented on the extract from conversations I had with Bell. I quote:

" 3. On at least one occasion it was agreed that we all believed statements like "The book in a drawer is still there even when not being observed" to be true. If some versions of physics did not agree, whether as accidental fact or by the definition of physics, too bad for those versions.
4. The experimental results disproved the EPR thesis, and implied superluminality."
**

It seems to me there are plenty of possibilities to keep nature local in a reasonable way without having to accept a terrible non-local theory.

Careful
 
Last edited:
  • #198
Ernies said:
All the arguments seem to be --- as I earlier remarked ---- descending to disagreements on terminology and in particular ontology. This, in the Oxford companion to Philosophy, is described as concerning (among other things) 'the nature of existence and the categorial structure of reality'. If that is true there cannot be a theory with no underlying ontology. Even the purported statement that 'there is no ontology' is itself ontological, since it categorises 'reality ' in denying it.

I completely agree with you. The existence of an external reality is a philosophic axiom in the sense of Aristotle: that it is true is available to direct sense experience (just open your eyes and you see... external reality!); and that this truth is an axiom is proved by the fact that any attempt to *deny* it is self-refuting.



Please come back to layman's language. It will help to clear the air. For example , so far as I know, no-one commented on the extract from conversations I had with Bell. I quote:

" 3. On at least one occasion it was agreed that we all believed statements like "The book in a drawer is still there even when not being observed" to be true. If some versions of physics did not agree, whether as accidental fact or by the definition of physics, too bad for those versions.
4. The experimental results disproved the EPR thesis, and implied superluminality."

Which bits of that do the contestants disagree with and why?

I agree with all of this (and with extremely minor and extremely rare exception, everything Bell ever said).

In simple terms, the upshot of this whole discussion is that nonlocality is real. Out there in external physical reality, things influence each other -- and, contrary to what we all expected since 1905, some of those influences propagate faster than the speed of light.

All of the attempts to deny this come down to either (a) there is no external reality, or (b) there is an external reality but it's nothing at all like what we thought (based on such evidence as the direct testimony of our eyes) it was like. That is, the only way to deny the reality of faster-than-light causation is to do something like what you allude to in 3 above, to advocate some version in which the book isn't there when you stop looking, or isn't really there even when you are looking and see it! And to that, you and I and Bell and all clear-thinking people say: too bad for those versions.

I have only one quibble with what you said in #4. The experimental results do indeed refute something, but it's not clear what you mean by "the EPR thesis". Usually this means something like: a deterministic ("hidden variable") replacement for Copenhagen QM. If that's what you meant, then your statement is misleading and/or wrong. It's true that the experiments refute this thesis, but that's hardly interesting by itself. What makes it interesting is remembering the *argument* that EPR gave *for* this thesis -- which is that this kind of theory is *required* by locality. It was only because they insisted on locality that EPR thought they could prove the thesis in question (local deterministic hidden variables). Bell said (and I agree): "My own first paper on this subject [Bell's Theorem] ... starts with a summary of the EPR argument *from locality to* deterministic hidden variables. But the commentators have almost universally reported that it begins with deterministic hidden variables." Hence, what turns out to be refuted by the experiments is not "the EPR thesis" per se (or at least that's not the *interesting* thing that's refuted by the experiments), but rather the *premise* from which (via the EPR argument) that thesis *followed*. And that is: locality, local causality, the idea that there are no faster-than-light causal influences out there in the world.

Is that sufficiently clear and non-cotton-candy-ish for you (and Einstein and Feynman)? :smile:


Clearly a Copenhagen adherent ( a breed I thought extinct) would call the para3 meaningless. But that is dogma; not an argument.

It's true, there aren't many Copenhagen dinosaurs left. But Copenhagen didn't (apparently) have a monopoly on nonsense. See, for example, Rovelli and "relational QM" (which posits that we each invent our own reality inside our heads... well, except that our heads are physical objects and hence just another mere subjective fantasy).
 
  • #199
Ernies said:
All the arguments seem to be --- as I earlier remarked ---- descending to disagreements on terminology and in particular ontology.

...

For example , so far as I know, no-one commented on the extract from conversations I had with Bell. I quote:

" 3. On at least one occasion it was agreed that we all believed statements like "The book in a drawer is still there even when not being observed" to be true. If some versions of physics did not agree, whether as accidental fact or by the definition of physics, too bad for those versions.
4. The experimental results disproved the EPR thesis, and implied superluminality."

Which bits of that do the contestants disagree with and why? Clearly a Copenhagen adherent ( a breed I thought extinct) would call the para3 meaningless. But that is dogma; not an argument.

Ernies

I agree that it comes back to terminology & definition in many cases. To address your specific questions about 3. and 4.:

3. If you assume that the observable value is there when there is no matching observation: then you more or less must reject EPR's definition of "element of reality". That is, the observable IS the reality. That means that every possible spin observation is an element of reality (i.e. an infinite number). How is that less baggage than MWI? I believe that reality is shaped by observation. Ergo, particle attributes are well-defined as a result of an observation, and not before. This view is fully consistent with the facts. I wish to point out that I am not denying the existence of the particle itself when it is not observed. I am simply denying that there are well defined values for the things we call observables independent of the observation.

4. I deny that superluminality is a rigid logical deduction from experimental results. But I can certainly understand why someone would tend to see non-locality from those same results.

As to the Copenhagen dinosaurs: how do you tell the difference between a physicist who is one and a physicist who is not? Only by their words, certainly not by their actions... :tongue:
 
  • #200
ttn said:
I completely agree with you. The existence of an external reality is a philosophic axiom in the sense of Aristotle: that it is true is available to direct sense experience (just open your eyes and you see... external reality!); and that this truth is an axiom is proved by the fact that any attempt to *deny* it is self-refuting.

It's true that the experiments refute this thesis, but that's hardly interesting by itself. What makes it interesting is remembering the *argument* that EPR gave *for* this thesis -- which is that this kind of theory is *required* by locality. It was only because they insisted on locality that EPR thought they could prove the thesis in question (local deterministic hidden variables). Bell said (and I agree): "My own first paper on this subject [Bell's Theorem] ... starts with a summary of the EPR argument *from locality to* deterministic hidden variables. But the commentators have almost universally reported that it begins with deterministic hidden variables." Hence, what turns out to be refuted by the experiments is not "the EPR thesis" per se (or at least that's not the *interesting* thing that's refuted by the experiments), but rather the *premise* from which (via the EPR argument) that thesis *followed*. And that is: locality, local causality, the idea that there are no faster-than-light causal influences out there in the world.

Is that sufficiently clear and non-cotton-candy-ish for you (and Einstein and Feynman)? :smile:

Yes, Indeed! Non-cotton-candy! I was aware of the basic thesis, but so many people are not. But you must remember what I said, I was trying to keep it to the simplest possible exposition. Thanks fo putting it in the open.

Ernies
 
  • #201
Ernies said:
Even the purported statement that 'there is no ontology' is itself ontological, since it categorises 'reality ' in denying it.
I disagree. I view an ontology as a mapping from our formal mathematical theory onto something we call "reality". As such, there's a big difference between saying that there is no ontology, and that the ontology maps onto "nothing".

Compare with the phrase "R doesn't have area" meaning that R is unmeasurable, versus "R has area zero" meaning that R is measurable, and has measure zero.

When I think of "no ontology", I interpret that as meaning that we simply don't attempt to map our formal structure onto some "element of reality" (or maybe even that no such map exists!). So, while denying reality means we must deny ontology1, the converse does not hold.



Ernies said:
" 3. On at least one occasion it was agreed that we all believed statements like "The book in a drawer is still there even when not being observed" to be true. If some versions of physics did not agree, whether as accidental fact or by the definition of physics, too bad for those versions.
It sounds like a statement of historical fact. (Or at least historical generalization) I suppose you mean if we should still agree with the sentiment? It's hardly conclusive.

My general preference is that "Is the book in the drawer?" isn't a well-posed question, unless you implicitly have some operational definition in mind... and the answer would depend on the particular theory and operational definition you are using.

Ernies said:
4. The experimental results disproved the EPR thesis, and implied superluminality."
And I would say that (at least the second half of) this is certainly wrong. You don't have:

(experimental results) ===> (superluminality)

you have

(experimental results) and (other assumptions) ===> (superluminality)

So, this implication is conditional on you accepting those other assumptions. Well, it's probably too harsh to say that Bell was wrong -- just that his conclusion is based upon assumptions he is making.



In regards to one of my earlier comments...
ttn said:
The existence of an external reality is a philosophic axiom in the sense of Aristotle: that it is true is available to direct sense experience (just open your eyes and you see... external reality!); and that this truth is an axiom is proved by the fact that any attempt to *deny* it is self-refuting.
The refutation to this argument has been around just as long as the argument itself, I believe. We, and our senses, are fallible. As such, we certainly cannot rely on them as a source of absolute truth!

And even if

(1) we can rely upon our senses and brains as sources of absolute truth
(2) we believed there was an external reality
(3) we manage to avoid coloring our perceptions with the biases we've accumulated through the years

you're still stuck with the problem that you aren't looking at external reality itself, but only the results of filtering it through our senses and preprocessing it with our brains.

(and I would argue that (3) is generally a much more problematic assumption than (1) :tongue:)



1: unless we are working with an empty mathematical structure! :smile:
 
  • #202
** Hence, what turns out to be refuted by the experiments is not "the EPR thesis" per se (or at least that's not the *interesting* thing that's refuted by the experiments), but rather the *premise* from which (via the EPR argument) that thesis *followed*. And that is: locality, local causality, the idea that there are no faster-than-light causal influences out there in the world. **

:frown: I have at least offered three more ways out than you, it seems that you basically either do not know these alternatives (by smart people), or you simply wish not to see, either acknowledge them. The problem with this entire discussion is that it is stigmatized around a *theorem* while physics is always about searching for possibilities beyond it. An equal logical conclusion from Bell's theorem is that the results of experiments aren't dichotomic variables ... and that statistics is relative. But of course, I assume that you as a Bohmian (the first ansatz anyone deduces for himself when thinking about one particle dynamics) *has* to deny these possibilities (btw I will come back in more detail where the argumentation in the paper you quoted puts in the ``solution'' of the measurement problem merely by hand) - you just want to see non-locality (at all cost). And I have to contradict Ernies when he states that people seem to have forgotten the conclusions you draw, these are simply the most standard textbook things around - which anyone knows and can be circumvented ! For example, Rafael Sorkin draws an entirely different conclusion in his quantum measure theory (the preclusion interpretation of QM) than Bell did.
 
Last edited:
  • #203
Ernies said:
Please come back to layman's language. It will help to clear the air.
" 4. The experimental results disproved the EPR thesis, and implied superluminality."

In layman's language the term local would mean a solution completed by the EPR goal of defining a “local deterministic hidden variable” as tnn has described it. Not in some alternate view of local supposed in an alternate external physical reality. Requiring such an alternate reality beyond our layman’s view of reality is essentially a non-local solution.

If EPR & locality is correctly disproved by the Bell Theorem I see no possibility to keep nature local other than to redefine the term local (away from the simple layman’s definition) in one that only exists due to an alternate external physical reality. We just have a very difficult time describing that “alternate reality” in terms of our layman’s reality. It’s hard to completely describe such a “background” on the 3D blackboard of our layman’s observed existence.

So I agree with tnn, the upshot of current evidence is that non-locality is real.
Any theory that successfully completes the picture of reality consistent with current evidence will necessarily be non-local.
Unless Bell is wrong and a local theory can produce a ‘local deterministic hidden variable’ to resolve the entanglement / superposition problem then only non-local theories (terrible or not) can be viable.

For a theory to truly be local as understood here; it must produce, define and describe such a ‘local deterministic hidden variable’.
 
  • #204
**
Unless Bell is wrong and a local theory can produce a ‘local deterministic hidden variable’ to resolve the entanglement / superposition problem then only non-local theories (terrible or not) can be viable. ***

Bell's theorem is correct but people only choose a narrow subset of the remaining possiblities to proceed; it is like Bell's view on local causality + no conspiracy + dichotomic outcome is holy, the only imaginable thing to do in a relativistic spirit ! That is the sad thing of this discussion and I as relativist can certainly see other possibilities. Also Aharonov has been playing around with the arrow of time, in order to offer a different view on this (you speak about all this as if he should be a crackpot, he must have been a complete lunatic no, to think about all this ? :frown: ). It seems likely to me that a reasonable combination of these alternatives can lead to a theory which is spacetime local in reasonable sense and satisfies perfectly experimental requirements.

Careful
 
Last edited:
  • #205
Careful said:
It seems likely to me that a reasonable combination of these alternatives can lead to a theory which is spacetime local in reasonable sense and satisfies perfectly experimental requirements.
To be clear; you expect a combination of "other possibilities" to produce a theroy that will give a ‘local deterministic hidden variable’ understandable in layman's terms.

I don't see how changing the arrow of time would retain local in these simple terms.
 
  • #206
**To be clear; you expect a combination of "other possibilities" to produce a theroy that will give a ‘local deterministic hidden variable’ understandable in layman's terms. **

I think you at least understand that Bell local and no conspiracy are two different things - the latter hypothesis which is very likely wrong (as progress in chaotic classical mechanics is suggesting).

**
I don't see how changing the arrow of time would retain local in these simple terms. **

Right, but there are ways out of that too (not that I favor this option, I simply mentioned it because it is a possibility). Certainly, it seems not plausible to me that this is the ONLY ingredient (provided I assume for a moment perfect measurement apparati and no signalling FTL) since then you would have a terrible non-locality too. Too make it more local, one immediately encounters the non-compactness dragon of the Lorentz group (null intervals in particular), a possible ``cure'' could be the introduction of a preferred frame and a (Riemannian) cutuff; that is the world is non-local but only on such and such scale... (this is of course the most simplistic thing I say here). A more daring proposition is that the basic laws of physics have nothing to do with Lorentzian geometry at all, that it all is some kind of Eulerian fluid dynamics (with time as a fifth dimension ??) and that Lorentzian symmetry is merely emergent for ``low'' energies, I cited many times Holland's paper - who does exactly that for the EM field - in that respect (an upgraded version of the good old vortex idea if you want to). But how you turn or twist the monster, you will have to show us why good old Albert was not a fool in assuming that the world is locally causal (but possibly conspirational - you see even GR can violate the assumptions behind Bell's theorem) on ``macroscopic'' scales.

Anyway, I remain open to many other scenario's - as you notice I am very keen on being able to make the link between cause and effect which has always been the longstanding goal of science.

Careful
 
Last edited:
  • #207
Hurkyl said:
I disagree. I view an ontology as a mapping from our formal mathematical theory onto something we call "reality". As such, there's a big difference between saying that there is no ontology, and that the ontology maps onto "nothing"...
Compare with the phrase "R doesn't have area" meaning that R is unmeasurable, versus "R has area zero" meaning that R is measurable, and has measure zero.....
When I think of "no ontology", I interpret that as meaning that we simply don't attempt to map our formal structure onto some "element of reality" (or maybe even that no such map exists!). So, while denying reality means we must deny ontology1, the converse does not hold...

And I would say that (at least the second half of) this is certainly wrong. You don't have:
(experimental results) ===> (superluminality)
you have
(experimental results) and (other assumptions) ===> (superluminality)
So, this implication is conditional on you accepting those other assumptions. Well, it's probably too harsh to say that Bell was wrong -- just that his conclusion is based upon assumptions he is making...

In regards to one of my earlier comments...
The refutation to this argument has been around just as long as the argument itself, I believe. We, and our senses, are fallible. As such, we certainly cannot rely on them as a source of absolute truth!

As I said half the argument is about terminology. At least you have stated your disagreement with the normal use of ontology clearly.

When you say "R is is unmeasurable" do you mean simply that the word 'area' is not applicable to R? Are you defining 'measure zero' as the end of an unending process such as ' as R tends to zero'? Otherwise I know it only as an arbitrarily defined reference point on an arbitrary scale, or a complete absence e.g. I have no [zero] oranges. I also have no antennae
and many other things, but it would be unwise to deduce much from that.

I have never maintained that our senses in themselves are a source of absolute truth. As a character (Dr. McPhee) in one of Lewis's novels says, in broad Doric "It's nae so much the facts, as the interrrrpretation thereof". At least we can agree on that, if not on the interpretation process. We do agree that an experimental result without other information/assumptions/theory is valueless, don't we?

Ernies
 
Last edited by a moderator:
  • #208
Ernies: R is unmeasurable when you're trying to measure the area of a gallon. It just doesn't compute. Like the location of a photon.

ttn: But Copenhagen didn't (apparently) have a monopoly on nonsense. See, for example, Rovelli and "relational QM". I thought it was interesting. It offers, ooh, I don't know, "Quantum Relativity". A lot of things are relative. Like energy. Like position. And if I close my eyes and try to feel the location of a magnet with another magnet, the feel I get depends on the magnet I'm holding.

All: what's this argument about? I've lost the plot. Not that I ever had it in the first place, but hey, I think we're over the hump with those particles.
 
  • #209
Farsight said:
Ernies: R is unmeasurable when you're trying to measure the area of a gallon. It just doesn't compute. Like the location of a photon.

The examples you give illustrate my point.
1. 'the area of a gallon' is an example of the property
'area' being totally inapplicable to the 'gallon': it is a logical impossibility.
2. I do not understand 'It just doesn't compute'. What computation? Or is this some patois for ' It doesn't make sense'. If so, I agree.
3. the 'Location of a photon' is completely different from 1. above. It is logically possible that the photon might have a position--- i.e. one has to show that it has not. The statement in itself makes sense: whether it is true is another matter.

It is precisely because of this type of confusion that I was asking questions

Ernies
 
  • #210
Farsight said:
ttn: But Copenhagen didn't (apparently) have a monopoly on nonsense. See, for example, Rovelli and "relational QM". I thought it was interesting. It offers, ooh, I don't know, "Quantum Relativity". A lot of things are relative. Like energy. Like position. And if I close my eyes and try to feel the location of a magnet with another magnet, the feel I get depends on the magnet I'm holding.

If you thought it was interesting, you probably didn't understand it. According to the theory put forward in that paper, two people can both look in Ernies drawer (not drawers!) and one of them can see the book there and one of them can fail to see the book there -- and they're both right, because the book *is* there "relative to the first observer" and the book *isn't* there "relative to the second observer". Anyone who thinks this makes sense has something in common with Bill Clinton: they don't know what the meaning of the word "is" is.
 
<h2>1. What is the holographic principle in quantum mechanics?</h2><p>The holographic principle is a concept in quantum mechanics that suggests that all the information about a three-dimensional space can be encoded on a two-dimensional surface. This means that the entire universe can be seen as a hologram, with the information about its three-dimensional structure stored on its boundary.</p><h2>2. How can we test the holographic principle?</h2><p>There are several ways to test the holographic principle, including using black hole physics, quantum entanglement, and quantum field theory. One proposed experiment involves using a high-energy particle accelerator to create a tiny black hole and observe its properties, which could provide evidence for the holographic principle.</p><h2>3. What is nonlocality in quantum mechanics?</h2><p>Nonlocality is a phenomenon in quantum mechanics where two or more particles can become entangled and their states become correlated, even if they are separated by large distances. This means that measuring the state of one particle can instantly affect the state of the other, regardless of the distance between them.</p><h2>4. How can we test nonlocality in quantum mechanics?</h2><p>One way to test nonlocality is through the Bell's inequality experiment, which involves measuring the correlations between entangled particles in different locations. If the results violate Bell's inequality, it would provide evidence for nonlocality in quantum mechanics.</p><h2>5. What are some challenges in testing the holographic principle and nonlocality in quantum mechanics?</h2><p>Some challenges in testing these concepts include the difficulty in creating and observing entangled particles, the complexity of quantum systems, and the need for advanced technology and techniques. Additionally, there are still many unanswered questions and debates surrounding these principles, making it challenging to design experiments that can definitively prove or disprove them.</p>

1. What is the holographic principle in quantum mechanics?

The holographic principle is a concept in quantum mechanics that suggests that all the information about a three-dimensional space can be encoded on a two-dimensional surface. This means that the entire universe can be seen as a hologram, with the information about its three-dimensional structure stored on its boundary.

2. How can we test the holographic principle?

There are several ways to test the holographic principle, including using black hole physics, quantum entanglement, and quantum field theory. One proposed experiment involves using a high-energy particle accelerator to create a tiny black hole and observe its properties, which could provide evidence for the holographic principle.

3. What is nonlocality in quantum mechanics?

Nonlocality is a phenomenon in quantum mechanics where two or more particles can become entangled and their states become correlated, even if they are separated by large distances. This means that measuring the state of one particle can instantly affect the state of the other, regardless of the distance between them.

4. How can we test nonlocality in quantum mechanics?

One way to test nonlocality is through the Bell's inequality experiment, which involves measuring the correlations between entangled particles in different locations. If the results violate Bell's inequality, it would provide evidence for nonlocality in quantum mechanics.

5. What are some challenges in testing the holographic principle and nonlocality in quantum mechanics?

Some challenges in testing these concepts include the difficulty in creating and observing entangled particles, the complexity of quantum systems, and the need for advanced technology and techniques. Additionally, there are still many unanswered questions and debates surrounding these principles, making it challenging to design experiments that can definitively prove or disprove them.

Similar threads

Replies
3
Views
980
Replies
1
Views
1K
  • Quantum Physics
Replies
6
Views
1K
  • Quantum Physics
Replies
28
Views
2K
  • Quantum Interpretations and Foundations
2
Replies
37
Views
1K
Replies
9
Views
1K
  • Quantum Physics
Replies
7
Views
3K
  • Quantum Interpretations and Foundations
15
Replies
491
Views
26K
Replies
43
Views
4K
Back
Top